Integral Test

turophile

Junior Member
Joined
May 22, 2010
Messages
94
I'm working on problems where I need to decide whether a series converges or diverges using the integral test. I've come across a series with a function I can't figure out how to integrate:

? 1/(ln x)[sup:34jfprlw]3[/sup:34jfprlw]

Is there a trick with this one where I should be able to see whether the series converges or diverges without actually evaluating the improper integral? Should I compare it with another (easier to integrate) series? Any help getting started would be appreciated.
 
This is not integrable in the usual sense (elementary methods).

This is known as the Logarithmic Integral. It has its own special name.

\(\displaystyle L_{i}(x)=\int_{2}^{x}\frac{1}{ln(u)}du\)

You see this with regards to higher math and advanced functions.

So, per Mathematica, \(\displaystyle \int\frac{1}{ln^{3}(x)}dx=\frac{1}{2}\left(L_{i}(x)-\frac{x(ln(x)+1)}{ln^{2}(x)}\right)\)

Incidentally, it diverges. \(\displaystyle \int_{2}^{\infty}\frac{1}{ln^{3}(x)}dx=\infty\)


Are you sure it is not

\(\displaystyle \sum_{k=2}^{\infty}\frac{1}{kln(k)^{3}}\)

That is a different matter. It can be integrated. Besides, this is a common problem in most calc texts when one gets to the integral test
 
I double checked, and the series in the problem is ? 1/(ln x)[sup:1adkg483]3[/sup:1adkg483]. Up to this chapter, my book has covered the comparison test, the ratio test, the ratio-comparison test, and L'Hopital's Rule, so I'm going to try some combination of those with the integral test to see where it takes me.
 
You can just use a comparison test.

\(\displaystyle \frac{1}{(ln(k))^{3}}>\frac{1}{k}\)

Since the harmonic series, \(\displaystyle \sum\frac{1}{k}\), diverges so does:

\(\displaystyle \sum_{k=2}^{\infty}\frac{1}{(ln(k))^{3}}\)

\(\displaystyle \frac{1}{(ln(k))^{3}}>\frac{1}{k}\) for large k.

Sorry to ramble in the last post, but I thought you HAD to us the integral test. Not a good choice for this one.
 
Going a bit further into later problems in this chapter, it's become clear to me that the authors wanted to demonstrate when *not* to use the integral test by giving this series to be evaluated for convergence/divergence *before* and *after* introducing Cauchy's condensation test.

If I'm using the condensation test rightly:

The general term of the condensed series is a[sub:3g6t7w19]n[/sub:3g6t7w19] = 2[sup:3g6t7w19]n[/sup:3g6t7w19]/(n * ln 2)[sup:3g6t7w19]3[/sup:3g6t7w19] = 1/(ln 2)[sup:3g6t7w19]3[/sup:3g6t7w19] * 2[sup:3g6t7w19]n[/sup:3g6t7w19]/n[sup:3g6t7w19]3[/sup:3g6t7w19]. Applying L'Hopital's Rule a few times, I'm looking at the limit as n approaches infinity of 1/(ln 2)[sup:3g6t7w19]3[/sup:3g6t7w19] * [n * (n - 1) * (n - 2) * 2[sup:3g6t7w19]n - 3[/sup:3g6t7w19]]/6, which diverges. Thus so does ? 1/(ln x)[sup:3g6t7w19]3[/sup:3g6t7w19].

Thanks for the comparison test example, galactus.
 
Top